Mathcenter Forum

Mathcenter Forum (https://www.mathcenter.net/forum/index.php)
-   ข้อสอบในโรงเรียน ม.ปลาย (https://www.mathcenter.net/forum/forumdisplay.php?f=21)
-   -   สมาคมคณิตศาสตร์ ม.ปลาย 2555 (https://www.mathcenter.net/forum/showthread.php?t=17761)

~ArT_Ty~ 25 พฤศจิกายน 2012 14:37

สมาคมคณิตศาสตร์ ม.ปลาย 2555
 
วันนี้สอบกลับมาเป็นไงกันบ้างครับ :)

มาแชร์คำตอบกับความคิดเห็นเกี่ยวกับข้อสอบปีนี้กันหน่อยฮะ

ใครมีโจทย์ช่วยเอามาลงให้หน่อยได้มั้ยครับ

คือว่าของผมทดจนเละไปแล้ว TT

-----------------
ดาวน์โหลด ข้อสอบสมาคม ม.ปลาย 2555

polsk133 25 พฤศจิกายน 2012 14:53

อะไรมันจะยากขนาดนี้ ม.ปลาย :blood::blood::blood:

gnap 25 พฤศจิกายน 2012 14:56

ได้ยินข่าวว่าออกให้ตัวแทนประเทศทำ-..-

tonklaZolo 25 พฤศจิกายน 2012 15:40

#เค้าบอกห้ามคัดลอก งั้นเขียนเรียงข้อเลยละกัน
1. ถ้าจุดโฟกัสทั้งสองของวงรี $\frac{x^2}{a^2}+\frac{y^2}{7}=1$ เป็นจุดเดียวกันกับจุดโฟกัสทั้งสองของไฮเปอร์โบลา $\frac{x^2}{144}-\frac{y^2}{81}=\frac{1}{25}$ แล้ว $a^2$ มีค่าเท่าใด
2. กำหนดให้ $A$ และ $B$ เป็นเหตุการณ์โดยที่ $P(A^c)=\frac{5}{6}$ และ $P(A^c\cap B^c)=\frac{1}{2}$ แล้ว $P(A-B)$ มีค่าเท่าใด
3. ถ้า $a_1,a_2,a_3,...$ เป็นลำดับเลขคณิตของจำนวนจริงโดยที่ $a_1\not= 0$ และ $a_{32}=4a_{20}$ แล้ว $a_{10}+a_{25}+2a_{40}$ มีค่าเท่ากับ $a_m$ แล้ว $m$ มีค่าเท่าใด
4. กำหนดให้ $\overrightarrow{A}=\bmatrix{a \\ b} $ และ $\overrightarrow{B}=\bmatrix{1 \\ \sqrt{3}} $ โดยที่ $b\not= 0$ และ $\overrightarrow{A}$ เป็นเวกเตอร์หนึ่งหน่วย ถ้ามุมระหว่าง $\overrightarrow{A}$ กับ $\overrightarrow{B}$ เท่ากับ $60^o$ แล้ว $a$ มีค่าเท่าใด
5. ถ้า $f(x)$ เป็นปฏิกิริยานุพันธ์ของ $\sqrt{x^3+1+2x\sqrt{x}}$ $\,$ แล้ว $f(1)-f(0)$ มีค่าเท่าใด
6. ถ้า $a$ และ $b$ เป็นคำตอบของสมการ $3^{2x+1}+2^{x+1}=6^x+2(3^{x+1})$ โดยที่ $a\not= b$ $\,$ แล้ว $\left(\,\frac{3}{2}\right)^{ab} \,$ มีค่าเท่าใด

ช่วยลงต่อกันด้วยนะคร๊าบ

lnพwsะบุ๑sสุ๑xล่o 25 พฤศจิกายน 2012 16:41

ข้อ 25 ได้เท่าไรกันครับ

nooonuii 25 พฤศจิกายน 2012 17:12

อ้างอิง:

ข้อความเดิมเขียนโดยคุณ TU Gifted Math#10 (ข้อความที่ 150921)
16. ให้ $I$ แทนเซตของจำนวนเต็มทั้งหมด ถ้า $f:I\rightarrow I$ โดยที่ $f(x+f(y))=x+y-4$ ทุกจำนวนเต็ม $x$ และ $y$ แล้ว $f(10)$ มีค่าเท่ากับเท่าใด
20. ถ้า $x,y$ และ $z$ เป็นจำนวนจริงที่ทำให้ $(4^{2x}+2)(4^{2y}+4)(4^{2z}+8)=4^{x+y+z+3}$ แล้ว $4^{2x+3y+4z}$ มีค่าเท่ากับเท่าใด

มีโจทย์แนวโอลิมปิกเพิ่มเข้ามามากก็ไม่ดีนะครับ ใช่ว่าผู้เข้าสอบทุกคนจะมีความรู้พวกนี้

16. $f(x)=x-2$
แทน $y=0$ และแทน $x$ ด้วย $x-f(0)$
หา $f(0)$ ได้ก็จบ

20. AM-GM ทีละก้อนจะได้ $x=\dfrac{1}{4},y=\dfrac{1}{2},z=\dfrac{3}{4}$

~ArT_Ty~ 25 พฤศจิกายน 2012 17:46

ข้อ 25 Hint : $1+\cos x = 2\cos ^2 x,\sin 2x=2\sin x\cos x$

Persister 25 พฤศจิกายน 2012 18:36

ข้อ 12 เลือกตอบได้เป็น 0,1,2 และ 3 เพื่อนๆ พี่ๆ น้องๆ ตอบอะไรกันบ้าง

Keng_Math 25 พฤศจิกายน 2012 18:47

ข้อ12 ตอบ0ป่ะครับ ใช้พวกlogลองแทนๆดู
ข้อ15 คิดยังไงครับ?

Thgx0312555 25 พฤศจิกายน 2012 19:01

ข้อ 25 ใช้ de mouivre // $cis(-\theta)=\dfrac{1}{cis\theta}$ ก็ได้ครับ

nooonuii 25 พฤศจิกายน 2012 20:00

อ้างอิง:

ข้อความเดิมเขียนโดยคุณ TU Gifted Math#10 (ข้อความที่ 150921)
25. จงหาค่าของ $\left(\dfrac{1+\sin\left(\dfrac{\pi}{2555}\right)+i\cos\left(\dfrac{\pi}{2555}\right)}{1+\sin\left(\dfrac{\pi}{2555}\right)-i\cos\left(\dfrac{\pi}{2555}\right)}\right)^{2555}$ โดยที่ $i$ เป็นจำนวนเชิงซ้อน ซึ่ง $i^2=-1$

ให้ $z=\cos\left(\dfrac{\pi}{2555}\right)+i\sin\left(\dfrac{\pi}{2555}\right)$

จะได้ $\overline{z}=\dfrac{1}{z}$ และ $z^{2555}=-1$

$\left(\dfrac{1+\sin\left(\dfrac{\pi}{2555}\right)+i\cos\left(\dfrac{\pi}{2555}\right)}{1+\sin\left(\dfrac{\pi}{2555}\right)-i\cos\left(\dfrac{\pi}{2555}\right)}\right)^{2555}=\left(\dfrac{1+i\overline{z}}{1-iz}\right)^{2555}$

$~\quad\quad\quad\quad\quad\quad\quad\quad\quad\quad\quad\quad\quad\quad\quad\quad\quad=\left(\dfrac{i}{z}\right)^{2555}$

$~\quad\quad\quad\quad\quad\quad\quad\quad\quad\quad\quad\quad\quad\quad\quad\quad\quad=i$

nooonuii 25 พฤศจิกายน 2012 20:35

อ้างอิง:

ข้อความเดิมเขียนโดยคุณ TU Gifted Math#10 (ข้อความที่ 150920)
12. กำหนดให้ $r$ และ $s$ เป็นจำนวนตรรกยะบวกใดๆ และ $x$ และ $y$ เป็นจำนวนอตรรกยะบวกใดๆ จงพิจารณาข้อความต่อไปนี้
(1) $r^s$ เป็นจำนวนตรรกยะบวก
(2) $r^x$ เป็นจำนวนอตรรกยะบวก
(3) $y^s$ เป็นจำนวนอตรรกยะบวก
(4) $x^y$ เป็นจำนวนอตรรกยะบวก
จำนวนข้อความที่เป็นจริงจากข้อความทั้ง 4 ข้อความข้างต้นตรงกับข้อใดต่อไปนี้

1. เท็จ เช่น $r=s=\dfrac{1}{2}$

2. เท็จ เช่น $r=1,x=\sqrt{2}$

3. เท็จ เช่น $y=\sqrt{2},s=2$

4. เท็จ เช่น $x=\sqrt{2},y=2\log_2 3$

Keehlzver 25 พฤศจิกายน 2012 20:36

ขอบคุณมากเลยครับ 35 ข้อ จำกันออกมาได้หมด :great: :great:

passer-by 25 พฤศจิกายน 2012 20:46

ปีนี้ผมเห็นว่า ไม่ยากมาก แต่ภาพรวมมันทดเลขไม่ค่อยลื่นเท่าไหร่ครับ

ผมคิดว่า ถึงเป็นตัวแทน IMO ก็คงไม่ชอบสไตล์นี้
----------------------------------------------------------------------------
ผมสนใจข้อ 21 ที่หา locus ของจุด

Focus อยู่ที่ $ (0,\frac{1}{16})$ ดังนั้น สมการเส้นตรงที่ผ่านโฟกัส คือ $ y- \frac{1}{16} = mx $

ดังนั้น จุดตัดเส้นตรงกับพาราโบลา สอดคล้องกับ $ 4x^2 - mx - \frac{1}{16} = 0 $ ซึ่งทุกจำนวนจริง m จะมีจำนวนจริง x 2 จำนวนต่างกันมารองรับเสมอ ,say, $ x_1 ,x_2 $

เท่ากับว่า พิกัด midpoint คอร์ด คือ $ (\frac{x_1+x_2}{2} , 2(x_1^2 +x_2^2)) = (\frac{m}{8} , \frac{2m^2+1}{16}) $

(สมการครึ่งหลัง ใช้สูตรผลบวก ผลคูณรากสมการกำลังสอง)

ดังนั้น locus ของ midpoint กำกับด้วยสมการ $ y = 8x^2 +\frac{1}{16}$

-------------------------------------------------------------------------------------

ส่วนอีกข้อที่ผมสนใจ คือ ข้อ 33 ครับ (แต่ผมขอใช้ความรู้เกินหลักสูตรนะครับ)

Take $ x_n = 3 \sin\theta_n$ โดย $\theta_n$ อยู่ในจตุภาคที่ 1 (รวม 0 กับ 90 องศาด้วย)

เงื่อนไขโจทย์ จึงเทียบเท่ากับ $ \frac{\sin \theta_n}{\cos (\theta_{n+1}/2) +\sin (\theta_{n+1}/2) } \geq \frac{1}{\sqrt{2}} \Rightarrow \sin (\theta_n) \geq \sin(\theta_{n+1}/2 +\frac{\pi}{4}) \Rightarrow \theta_{n+1} \leq 2\theta_n - \frac{\pi}{2}$

อสมการสุดท้าย implies 2 อย่างคือ $ \theta_n$ เป็นลำดับไม่เพิ่ม (เพราะมุมจำกัดในจตุภาคที่ 1 )

และ ทุกมุมต้องมีค่าอย่างน้อย $ \frac{\pi}{4}$ ด้วย

Now ลำดับ $\theta_n $ bounded และ nonincreasing แสดงว่ามี limit ,say ,L และ $ L \leq 2L - \frac{\pi}{2}$

จากอสมการและ by nonincreasing แสดงว่า ลำดับนี้ เป็น constant sequence ลู่เข้าหา $ L= \frac{\pi}{2}$

และทำให้ $x_n =3 $

TU Gifted Math#10 25 พฤศจิกายน 2012 21:05

สมาคมคณิตศาสตร์แห่งประเทศไทย ในพระบรมราชูปถัมภ์
ข้อสอบแข่งขันคณิตศาสตร์ ประจำปีการศึกษา $2555$ ระดับมัธยมศึกษาตอนปลาย
สอบวันอาทิตย์ที่ $25$ พฤศจิกายน $2555$ เวลา $9:00-12:00$ น.[/b]
ตอนที่ 1
1. ถ้าจุดโฟกัสทั้งสองของวงรี $\frac{x^2}{a^2}+\frac{y^2}{7}=1$ เป็นจุดเดียวกันกับจุดโฟกัสทั้งสองของไฮเพอร์โบลา $\frac{x^2}{144}-\frac{y^2}{81}=\frac{1}{25}$ แล้ว $a^2$ มีค่าตรงกับข้อใด
2. กำหนดให้ $A$ และ $B$ เป็นเหตุการณ์โดยที่ $P(A^c)=\frac{5}{6}$ และ $P(A^c\cap B^c)=\frac{1}{2}$ แล้ว $P(B-A)$ ตรงกับข้อใดต่อไปนี้
3. ถ้า $a_1,a_2,a_3,...$ เป็นลำดับเลขคณิตของจำนวนจริงโดยที่ $a_1\not= 0$ และ $a_{32}=4a_{20}$ แล้ว $a_10+a_{25}+2a_{40}$ มีค่าตรงกับข้อใดต่อไปนี้
4. กำหนดให้ $ \vec{A}=\bmatrix{a \\ b} $ และ $\vec{B}=\bmatrix{1 \\ \sqrt{3}}$ โดยที่ $b\not= 0$ และ $\vec{A}$ เป็นเวกเตอร์หนึ่งหน่วย ถ้ามุมระหว่าง $\vec{A}$ กับ $\vec{B}$ เท่ากับ $60^{\circ}$ แล้ว $a$ มีค่าตรงกับข้อใดต่อไปนี้
5. ถ้า $f(x)$ เป็นปฏิยานุพันธ์ของ $\sqrt{x^3+1+2x\sqrt{X}}$ แล้ว $f(1)-f(0)$ มีค่าตรงกับข้อใดต่อไปนี้
6. ถ้า $a$ และ $b$ เป็นคำตอบของสมการ $3^{2x+1}+2^{x+1}=6^x+2(3^{x+1})$ โดยที่ $a\not= b$ แล้ว $(\frac{3}{2})^{ab}$ มีค่าตรงกับข้อใดต่อไปนี้
7. ถ้า $A$ เป็นเมทริกซ์จัตุรัสขนาด $2\times 2$ ซึ่ง $A^2=2(A+I_2)$ โดยที่ $I_2$ แทนเมทริกซ์เอกลักษณ์ขนาด $2\times 2$ แล้ว $\mid det(A-I_2)\mid$ มีค่าตรงกับข้อใดต่อไปนี้
8. กำหนดให้ $f(x)=x-\sqrt{x^2-1}$ และ $g(x)=\frac{1}{x+\sqrt{x^2-1}}$
จงพิจารณาข้อความต่อไปนี้
(1) โดเมนของ $f$ $=$ โดเมนของ $g$
(2) $f=g$
ข้อใดต่อไปนี้ถูกต้อง
9. เด็กนักเรียนมัธยมปลายห้องหนึ่งมีจำนวนทั้งสิ้น $a+b$ คน ประกอบด้วยเด็กนักเรียนชาย $a$ คน และเด็กนักเรียนหญิง $b$ คน ถ้าสุ่มเลือกนักเรียนมา $2$ คน จากนักเรียนทั้ง $a+b$ คนเหล่านี้ ปรากฏว่า ความน่าจะเป็นที่เด็กนักเรียนที่เลือกมา $2$ คนนี้เป็นเพศเดียวกันมีค่าเท่ากับ $\frac{1}{2}$ จงพิจารณาว่า $a^2+b^2-2ab$ มีค่าตรงกับข้อใดต่อไปนี้
10. ถ้า $x$ เป็นจำนวนจริงที่อยู่ในช่วง$[0,\frac{\pi}{2}]$ และ สอดคล้องกับสมการ $arcsin(cos x)+arccos(sin x)=1$ แล้ว $x$ เป็นสมาชิกของช่วงใดต่อไปนี้
11. กำหนดให้ $p$ และ $q$ เป็นจำนวนเชิงซ้อน โดยที่ $q\not= 0$ ซึ่งทำให้สมการ $z^2+pz+q=0$ มีรากที่ต่างกันเป็นจำนวนเชิงซ้อน $z_1,z_2$ ถ้า $\mid z_1 \mid =1=\mid z_2 \mid$ แล้ว ส่วนจริงของ $z_1\overline{z_2}$ มีค่าตรงกับข้อใดต่อไปนี้
12. กำหนดให้ $r$ และ $s$ เป็นจำนวนตรรกยะบวกใดๆ และ $x$ และ $y$ เป็นจำนวนอตรรกยะบวกใดๆ จงพิจารณาข้อความต่อไปนี้
(1) $r^s$ เป็นจำนวนตรรกยะบวก
(2) $r^x$ เป็นจำนวนอตรรกยะบวก
(3) $y^s$ เป็นจำนวนอตรรกยะบวก
(4) $x^y$ เป็นจำนวนอตรรกยะบวก
จำนวนข้อความที่เป็นจริงจากข้อความทั้ง 4 ข้อความข้างต้นตรงกับข้อใดต่อไปนี้
13. สำหรับ $X$ และ $Y$ ที่เป็นเมทริกซ์ขนาน $3\times 3$ ใดๆ นิยาม$[X,Y]=XY-YX$ ให้ $A,B,C$ และ $D$ เป็นเมทริกซ์ขนาด $3\times 3$ ใดๆ จงพิจารณาข้อความต่อไปนี้
(1) $det([A,B])=-det([B,A])$
(2) $[A+C,B+D]=[A,B]+[C,D]$
(3) $[A,[B,C]]+[B,[C,A]]=-[C,[A,B]]$
ข้อใดต่อไปนี้ถูกต้อง
14. กำหนดให้ $f$ เป็นฟังก์ชันจากเซตของจำนวนจริงไปยังเซตของจำนวนจริง และให้ $a$ เป็นจำนวนจริง จงพิจารณาข้อความต่อไปนี้
(1) ถ้า $ \textstyle\lim_{h\to 0}\frac{f(a+h)-f(a-h)}{h} =10$ แล้ว $ \textstyle\lim_{h\to 0}\frac{f(a+h)-f(a)}{h} =5$
(2) ถ้า $ \textstyle\lim_{h\to 0}\frac{f(a+h)-f(a)}{h} =5$ แล้ว $ \textstyle\lim_{h\to
0}\frac{f(a+h)-f(a-h)}{h} =10$
ข้อใดต่อไปนี้ถูกต้อง
15. กำหนดให้ $ABC$ เป็นสามเหลี่ยมใดๆ โดยที่ $cos^2 A+cos^2 B \geq sin^2 C$ ข้อใดต่อไปนี้ถูกต้อง

TU Gifted Math#10 25 พฤศจิกายน 2012 21:06

ตอนที่ 2
16. ให้ $I$ แทนเซตของจำนวนเต็มทั้งหมด ถ้า $f:I\rightarrow I$ โดยที่ $f(x+f(y))=x+y-4$ ทุกจำนวนเต็ม $x$ และ $y$ แล้ว $f(10)$ มีค่าเท่ากับเท่าใด
17. กำหนดให้ $A=\{x\in\mathbb{R}\mid\frac{1}{sin^2x}-\frac{1}{cos^2x}-\frac{1}{tan^2x}-\frac{1}{cot^2x}-\frac{1}{sec^2s}-\frac{1}{csc^2x}=-3\}$ และ $B=\{x\in A\mid 0<x<2\pi\}$ ผลบวกสมาชิกทั้งหมดของเซต $B$ มีค่าเท่ากับเท่าใด
18. นักเรียนมัธยมศึกษาตอนปลายห้องหนึ่งมีจำนวนทั้งหมด $63$ คน ประกอบไปด้วยนักเรียนชาย $a$ คน และนักเรียนหญิง $b$ คน ในการสอบวิชาคณิตศาสตร์ครั้งหนึ่งปรากฏว่า คะแนนเฉลี่ยนของนักเรียนชาย เท่ากับ $a$ คะแนน และคะแนนเฉลี่ยของนักเรียนหญิง เท่ากับ $b$ คะแนน ถ้าคะแนนเฉลี่ยของนักเรียนทั้งห้องเท่ากับ $35$ คะแนน แล้ว ค่า $b$ ที่เป็นไปได้ทั้งหมดเท่ากับเท่าใด
19. กำหนดให้ $ABC$ เป็นสามเหลี่ยมในระนาบพิกัดฉาก $XY$ โดยที่ $ABC$ มีความยาวเส้นรอบรูปเท่ากับ 36 หน่วย ให้ $L,M$ และ $N$ เป็นจุดบนด้าน $BC,AC$ และ $AB$ ตามลำดับ โดยที่ ด้าน $BL$ ยาว $3$ หน่วย ด้าน $CM$ ยาว $4$ หน่วย และด้าน $AN$ ยาว $5$ หน่วย ถ้า $\vec{AL}+\vec{BM}+\vec{CN}=0$ แล้ว $\vec{AB} \cdot \vec{AC}$ มีค่าเท่ากับเท่าใด
20. ถ้า $x,y$ และ $z$ เป็นจำนวนจริงที่ทำให้ $(4^{2x}+2)(4^{2y}+4)(4^{2z}+8)=4^{x+y+z+3}$ แล้ว $4^{2x+3y+4z}$ มีค่าเท่ากับเท่าใด
21. ให้ $P$ เป็นพาราโบลา $y=4x^2$ และ $F$ เป็นจุดโฟกัสของ $P$ จงหาสมการทางเดินของจุดกึ่งกลางของคอร์ดของ $P$ ซึ่งคอร์ดเหล่านี้ผ่านจุดโฟกัส $F$
22. ตารางขนาด $1\times 8$ ตารางหน่วย แบ่งเป็นช่องขนาด $1\times 1$ ตารางหน่วย จำนวน $8$ ช่อง ระบายสีแต่ละช่องด้วยสีเพียงหนึ่งสี จากสีแดง เหลือง ดำ โดยที่จำนวนช่องที่มีสีแดงเป็นจำนวนคี่ และตารางนี้ถูกระบายครบทุกสี จงหาจำนวนรูปแบบทั้งหมดที่เป็นไปได้จากการระบายสีตารางตามเงื่อนไขดังกล่าว
23. กำหนดให้ $l$ เป็นเส้นตรงที่มีความชัน $\frac{7}{2}$ และเส้นตรง $l$ ไม่ตัดกับพาราโบลา $y=x^2$ กำหนดให้ $A$ และ $B$ เป็นจุด $2$ จุดบนเส้นตรง $l$ โดยที่ ส่วนของเส้นตรง $AB$ ยาว $2555$ หน่วย จงหาพิกัดจุด $C$ ที่อยู่บนพาราโบลา $y=x^2$ ซึ่งทำให้สามเหลี่ยม $ABC$ มีพื้นที่น้อยที่สุด
24. กำหนดให้ $f:[-4,\infty)\rightarrow\mathbb{R}$ โดยที่ $f(x)=\frac{x^2+4x+3}{x^2+7x+14}$ จงหาเรนจ์ของฟังก์ชัน $f$
25. จงหาค่าของ $(\frac{1+sin(\frac{\pi}{2555})+icos(\frac{\pi}{2555})}{1+sin(\frac{\pi}{2555})-icos(\frac{\pi}{2555})})^{2555}$ โดยที่ $i$ เป็นจำนวนเชิงซ้อน ซึ่ง $i^2=-1$

TU Gifted Math#10 25 พฤศจิกายน 2012 21:06

ตอนที่ 3
26. กำหนดให้ $a_1,a_2,\ldots ,a_n$ เป็นลำดับของจำนวนเต็มบวกเรียงติดกันที่มากกว่า $1$ พจน์ (เช่น $78,79,80,81,82$) ถ้าผลรวมของพจน์ในทุกพจน์ในลำดับเท่ากับ $2012$ แล้ว ค่าของ $a_1$ เท่ากับเท่าใด
27. โจทย์ไม่สมบูรณ์ ยกผลประโยชน์ให้ผู้เข้าสอบ
28. มีไม้ขีดไฟ $10$ ก้านที่มีความยาว $1,2,\ldots ,10$ หน่วย ตามลำดับ สุ่มหยิบไม้ขีดมา $3$ ก้าน จงหาความน่าจะเป็นที่ไม้ขีดทั้งสามก้านสามารถประกอบเป็นด้านของรูปสามเหลี่ยมมุมป้านได้
29. ให้ $S$ เป็นอาณาบริเวณที่ปิดล้อมด้วยกราฟ $y=x(1-x)$ กับแกน $x$ จงหาค่า $m$ ทั้งหมดที่ทำให้เส้นตรง $y=mx$ แบ่ง $S$ เป็น $2$ ส่วนโดยที่แต่ละส่วนมีพื้นที่เท่ากัน
30.กำหนดให้ $A,B$ และ $C$ เป็นสับเซตของ $\{1,2,\ldots ,12\}$ โดยที่ $A\cap B\cap C=\{1\}$ , จำนวนสมาชิกของ $A,B$ และ $C$ มีค่าเท่ากัน และสอดคล้องกับสมบัติต่อไปนี้
(1) $\forall a\in A$ $\exists b\in B, a\mid b$
(2) $\forall b\in B$ $\exists c\in C, b\mid c^2$
(3) $\forall c\in C$ $\exists a\in A, c^2\mid a^3$
ในบรรดาเซต $A$ ทั้งหมดที่เป็นไปได้ $A$ มีผลบวกสมาชิกทั้งหมดมากที่สุดเท่ากับเท่าใด
31. กำหนดให้ $a,b$ และ $c$ เป็นจำนวนจริงที่ทำให้สมการ $asin(x-\frac{\pi}{4})+bsinx+csin(x+\frac{\pi}{4})=0$ มีคำตอบอย่างน้อยสองค่าในช่วง $(0,\pi)$ จงหาสามสิ่งอันดับ $(a,b,c)$ ทั้งหมดที่เป็นไปได้เมื่อ $a$ เป็นจำนวนเต็ม และ $1<b<5$
32. กำหนดให้ $A=\{(2x,2y)\in\mathbb{R}^2\mid\log_x (log_y x)>0\}$ และ $B=\{(x,y)\in\mathbb{R}^2\mid\mid x\mid+\mid y\mid<2555$ และ $x$ และ $y$ เป็นจำนวนเต็ม $\}$ จงหาจำนวนสมาชิกของ $A\cap B$
33. กำหนดให้ $x_1,x_2,\ldots,x_n,\ldots$ เป็นลำดับอนันต์ของจำนวนจริงในช่วง $[0,3]$ ซึ่งสอดคล้องกับอสมการ $\frac{x_n}{\sqrt{x_{n+1}+3}}\geq \sqrt{\frac{3}{2}}$ ทุก $n=1,2,3,\ldots$ ค่าของ $x_{2555}$ ทั้งหมดที่เป็นไปได้เท่ากับเท่าใด
34. สำหรับจำนวนจริง $x$ ใดๆ กำหนดให้ $[x]$ เป็นจำนวนเต็มที่มากที่สุดซึ่งมีค่าไม่เกิน $x$ จงหาว่า มีจำนวนเต็มบวก $n$ ทั้งหมดกี่จำนวนซึ่งทำให้ $\sum_{j=1}^{2555}[2^{-j}n] =n-1$
35. สำหรับแต่ละจำนวนเต็มบวก $n$ กำหนดให้ $a_n=$ จำนวนจริงที่มีค่ามากที่สุดที่ทำให้เส้นตรง $y=a_nx$ ตัดกราฟ $y=sin x$ ทั้งหมด $4n+1$ จุด จงหาค่าของ $\lim_{n\rightarrow\infty} na_n$

ข้อสอบชุดนี้เป็นลิขสิทธิ์ตามกฏหมายของสมาคมคณิตศาสตร์แห่งประเทศไทย ในพระบรมราชูปถัมภ์ ห้ามคัดลอกหรือทำสำเนาซ้ำ เฉลย และจำหน่ายโดยเด็ดขาด

~ArT_Ty~ 25 พฤศจิกายน 2012 21:44

ขอเฉลยข้อ 30 กับข้อ 31 หน่อยครับ

Thgx0312555 25 พฤศจิกายน 2012 22:33

31
จัดรูปได้ $(\dfrac{a}{\sqrt{2}}+b+\dfrac{c}{\sqrt{2}})sinx+(\dfrac{a}{\sqrt{2}}-\dfrac{c}{\sqrt{2}})cosx=0$
ซึ่งจะเขียนใหม่ได้อยู่ในรูป $Asin(x+\theta)=0$
ซึ่งถ้า $A \not= 0$ สมการจะมีไม่เกินคำตอบเดียวในช่วงที่กำหนดให้
$A=0$
นั่นคือ
$\dfrac{a}{\sqrt{2}}+b+\dfrac{c}{\sqrt{2}}=0$ และ
$\dfrac{a}{\sqrt{2}}-\dfrac{c}{\sqrt{2}}=0$
$(a,b,c)=(-1,\sqrt{2},-1),(-2,2\sqrt{2},-2),(-3,3\sqrt{2},-3)$

แต่ในห้องสอบผมตอบผิด = =

lnพwsะบุ๑sสุ๑xล่o 25 พฤศจิกายน 2012 22:38

คุณ Thgx0312555 ได้เยอะแล้วครับ :)

Thgx0312555 25 พฤศจิกายน 2012 22:51

30
ตัวเลขที่มีใน $A \cup B \cup C$ ได้ ได้แก่ 1 2 3 4 6 8 9 12
ใน A,B,C รวมกันจะมี 1 ได้ 3 ตัว 2,3,4,6,8,9,12 มีได้อย่างละ 2 ตัว
$n(A)+n(B)+n(C) \le 17$
$n(A) \le 5$
$A$ ซึ่งให้ผลรวมสมาชิกมากสุด คือ $\left\{ 1,6,8,9,12 \right\}$
ผลบวกมากที่สุดคือ $36$

ตัวอย่างการสร้าง A,B,C ที่สอดคล้อง
A = {12, 9, 8, 6, 1}
B = {12, 9, 8, 3, 1}
C = {6, 4, 3, 2, 1}

แม่ให้บุญมา 25 พฤศจิกายน 2012 23:01

ตอนที่ 3
28. มีไม้ขีดไฟ ก้านที่มีความยาว หน่วย ตามลำดับ สุ่มหยิบไม้ขีดมา ก้าน จงหาความน่าจะเป็นที่ไม้ขีดทั้งสามก้านสามารถประกอบเป็นด้านของรูปสามเหลี่ยมมุมป้านได้

p=n(E)/n(S) โดย n(S)=10C3=10!/(3!7!)=120
n(E) อาจต้องแจกแจง สามเหลี่ยมที่ประกอบด้วย ความยาวก้าน a,b,c
โดยมีเงื่อนไขในการเลือกค่า c ที่จะได้∆ มุมป้านจาก
a<b<c โดยที่ sqrt(a²+b²)<c<a+b หรือ
(a²+b²)<c²<(a+b)²
เช่น ถ้า a=3, b=4
(3²+4²)<c²<(3+4)²
25< c²<49 ได้ c=6
ถ้า a=3, b=5
(3²+5²)<c²<8²
ได้ c=6,7 (c²=36,7²) เป็นต้น
ถ้าไล่หา c โดย เริ่มจาก a=2,b=3
ถึง a=6,b=7 ได้ c=10 ค่าเดียวเป็นอันสุดท้าย
ลองไล่แล้วบวกหาจำนวนค่า c รวมได้เป็น n(E) =32
ตอบ p=32/120

~ArT_Ty~ 25 พฤศจิกายน 2012 23:26

อ้างอิง:

ข้อความเดิมเขียนโดยคุณ Thgx0312555 (ข้อความที่ 150997)
30
ตัวเลขที่มีใน $A \cup B \cup C$ ได้ ได้แก่ 1 2 3 4 6 8 9 12
ใน A,B,C รวมกันจะมี 1 ได้ 3 ตัว 2,3,4,6,8,9,12 มีได้อย่างละ 2 ตัว
$n(A)+n(B)+n(C) \le 17$
$n(A) \le 5$
$A$ ซึ่งให้ผลรวมสมาชิกมากสุด คือ $\left\{ 1,6,8,9,12 \right\}$
ผลบวกมากที่สุดคือ $36$

ตัวอย่างการสร้าง A,B,C ที่สอดคล้อง
A = {12, 9, 8, 6, 1}
B = {12, 9, 8, 3, 1}
C = {6, 4, 3, 2, 1}

ไม่เข้าใจอ่ะครับว่าทำไมต้องเอา 5,7,10,11 ออกไปอ่ะครับ?? :confused:

Mol3ilE 26 พฤศจิกายน 2012 00:06

ข้อ34 ตอบอะไรอ่ะครับ

Thgx0312555 26 พฤศจิกายน 2012 00:23

ถ้ามี 7, 11 ในเซตหนึ่ง โดยเงื่อนไขอีกสองเซตก็ต้องมีด้วย
ซึ่งขัดแย้งกับ $A \cap B \cap C = \left\{ 1 \right\}$

ทำนองเดียวกันกับ 10

ถ้ามี 5 ในเซตหนึ่ง โดยเงื่อนไขต้องอีกสองเซตต้องมีตัวหาร 5 ลงตัวด้วย
แต่ทุกเซตไม่มี 10 ดังนั้นทุกเซตต้องมี 5 จึงขัดแย้ง

ง่วงนอน 26 พฤศจิกายน 2012 08:06

เป็นข้อสอบที่ดีมากๆคร๊าบบบ

N'Pon Jukku AuHu Jealyai 26 พฤศจิกายน 2012 16:38

ใครมีข้อสอบจริงมั่งอ่า
อยากเหนข้อสอบจริงอ่า
้เสียตังแม่งร้อยนึง นศท เสือกนัดไปสอบวันนั้น
นัดทีหลังด้วย อดๆ

แม่ให้บุญมา 27 พฤศจิกายน 2012 02:58

ตอนที่ 2
18. นักเรียนมัธยมศึกษาตอนปลายห้องหนึ่งมีจำนวนทั้งหมด คน ประกอบไปด้วยนักเรียนชาย a คน และนักเรียนหญิง b คน ในการสอบวิชาคณิตศาสตร์ครั้งหนึ่งปรากฏว่า
คะแนนเฉลี่ยนของนักเรียนชาย เท่ากับ a คะแนน และคะแนนเฉลี่ยของนักเรียนหญิง เท่ากับ b คะแนน ถ้าคะแนนเฉลี่ยของนักเรียนทั้งห้องเท่ากับ 35 คะแนน แล้ว ค่า b ที่เป็นไปได้ทั้งหมดเท่ากับเท่าใด

สมการ คะแนนรวม
คะแนนเฉลียของห้อง x จำนวนนักเรียนทั้งหมด = คะแนนเฉลียของนักเรียนชาย จำนวนนักเรียนชาย+ คะแนนเฉลียของนักเรียนหญิงx จำนวนนักเรียนหญิง
35 x 63=axa +bxb=a²+b²=a²+(63-a)²
2a² -126a+1764=0 ได้ a=21,42 ในทำนองเดียวกันได้
b= 42 และ 21

No.Name 27 พฤศจิกายน 2012 05:06

#21 ผมงงอ่ะครับทำไม

ทำไม $A={1,12,9,8,6}$ แล้วมันไปหาร $12,9,8,3,1$ ลงตัวอ่ะครับ ไม่เข้าใจ

มันบอกว่า ทุกตัวใน A จะต้องไปหาร บางตัวใน B ไม่ใช่หรอครับ

bell18 27 พฤศจิกายน 2012 14:56

หน้าบอร์ดของ ม.ต้น โพสต์รูปข้อสอบลงไปเรียบร้อยแล้ว ของม.ปลายน่าจะโพสต์รูปข้อสอบลงไปบ้างนะครับ

Keehlzver 28 พฤศจิกายน 2012 01:20

ข้อ 33 เปลี่ยนเป็น cos ก็ได้เหมือนกัน แต่สมบัติจะตรงกันข้ามกับที่พี่ passer-by โพสต์

ปล. ข้อสอบปีนี้โหดมากเลยครับ

gon 28 พฤศจิกายน 2012 14:29

อ้างอิง:

ข้อความเดิมเขียนโดยคุณ bell18 (ข้อความที่ 151192)
หน้าบอร์ดของ ม.ต้น โพสต์รูปข้อสอบลงไปเรียบร้อยแล้ว ของม.ปลายน่าจะโพสต์รูปข้อสอบลงไปบ้างนะครับ

วันศุกร์นี้คาดว่าผมจะได้ข้อสอบฉบับเต็มมาครับ ถ้ายังไม่มีใครแปะฉบับเต็ม คาดว่าไม่เกินวันจันทร์ที่จะนี้ผมจะหาเวลา scan ให้

No.Name 28 พฤศจิกายน 2012 21:32

ข้อ 30 ยังไงอ่ะครับช่วยอธิบายหน่อยได้ไหมครับ ?

แม่ให้บุญมา 28 พฤศจิกายน 2012 22:26

ตอนที่ 2
ข้อ 16. ให้ $I$ แทนเซตของจำนวนเต็มทั้งหมด ถ้า $f:I\rightarrow I$ โดยที่ $f(x+f(y))=x+y-4$ ทุกจำนวนเต็ม $x$ และ $y$ แล้ว $f(10)$ มีค่าเท่ากับเท่าใด$

ลองให้ f(y)=ay+b จะได้
f[x+(ay+b)]=a[x+ay+b]+b=ax+a²y+ab+b= x+y-4
พิจารณาสัมประสิทธิของ x และ y จะได้ a=1→ b=-4/2=-2
ดังนั้น f(10)=1(10)-2=8

แม่ให้บุญมา 29 พฤศจิกายน 2012 15:26

ตอนที่ 3
33. กำหนดให้ $ x_1,x_2,\ldots,x_n,\ldots$ เป็นลำดับอนันต์ของจำนวนจริงในช่วง $[0,3]$ ซึ่งสอดคล้องกับอสมการ $\frac{x_n}{\sqrt{x_{n+1}+3}}\geq \sqrt{\frac{3}{2}}$
ทุก $n=1,2,3,\ldots$ ค่าของ $x_{2555}$ ทั้งหมดที่เป็นไปได้เท่ากับเท่าใด

จากโจทย์ได้ $2\times X_n^2 \ge 3[X_{n+1}+3]$
ได้รูปทั่วๆไป $X_{n+1} \le \frac{2X_n^2 - 9}{3}$
นี่ดูค่า X จะลดลงไปเรื่อยๆ
ถ้าเริ่มจากค่าสูงสุด $X_1=3$ จะไ้ด้ $X_2 \le \frac{[2(9)-9]}{3}=3 $หรือได้ $X_2=[0,3]$
ถ้า $X_1 \lt 3$ จะได้ $X_3 \lt X_2$ จะได้ $X_1 \gt X_2 \gt X_3 \gt X_4 \gt ...$นั่นคือจะได้ $X_{2555}=0$
สรุปถ้า$ X_1=3$ จะได้ $X_{2555}=[0,3]$
สรุปถ้า $X_1 \lt 3 $ จะได้ $ X_{2555}=0$
ถ้า $X_n=\frac{3}{\sqrt{2}} =2.12132 $ จะได้ $X_{n+1}=0 $ พอดี

แม่ให้บุญมา 30 พฤศจิกายน 2012 16:37

B]21.[/b] ให้ $P$ เป็นพาราโบลา $y=4x^2$ และ $F$ เป็นจุดโฟกัสของ $P$ จงหาสมการทางเดินของจุดกึ่งกลางของคอร์ดของ $P$ ซึ่งคอร์ดเหล่านี้ผ่านจุดโฟกัส $F$
จากสมการ พาราโบลา 4x²=y₁ หรือ x²=4(1/16)y₁=4cy₁
สมการเส้นตรงระหว่างจุดตัด พาราโบลา x₁,x₂ และผ่านจุดโฟกัส c=(0,1/16) คือ y₂-1/16 = mx เมื่อ m เป็นความชันของเส้นตรง
เมื่อ y₁=y₂ จะได้ 4x² - mx -1/16=0 →
x² - mx/4 -1/64= (x-x₁)(x-x₂)=x²-(x₁+x₂)x+ x₁x₂
เทียบสัมประสิทธิ์จะได้ x₁+x₂=m/4
แทนค่า x=(x₁+x₂)/2=m/8 (จุดกึ่งกลางของเส้นตรง ในสมการเส้นตรงได้
y₂= mx+1/16= m(m/8)+1/16=8(m/8)²+1/16=8x²+1/16 หรือ
x²=(1/8)(y-1/16)= 4(1/32)(y-1/16)
ได้ ทางเดินจุดกึ่งกลางเส้นตรงที่ผ่านจุดโฟกัสเดิมของพาราโบลาเป็นพาราโบลาที่มี ความยาวโฟกัส=1/32
มีจุดยอดอยู่ที่จุดโฟกัสเดิม คือที่ (0,1/16)

passer-by 01 ธันวาคม 2012 03:09

อ้างอิง:

ข้อความเดิมเขียนโดยคุณ TU Gifted Math#10 (ข้อความที่ 150977)
35. สำหรับแต่ละจำนวนเต็มบวก $n$ กำหนดให้ $a_n=$ จำนวนจริงที่มีค่ามากที่สุดที่ทำให้เส้นตรง $y=a_nx$ ตัดกราฟ $y=sin x$ ทั้งหมด $4n+1$ จุด จงหาค่าของ $\lim_{n\rightarrow\infty} na_n$

ตอนแรกไม่สนใจข้อนี้เท่าไหร่ แตตอนนี้ เห็นเงียบๆ ก็เลยขอลองทดดูครับ

ในฝั่งบวก กราฟจะตัด sin x ใน $ (0, \pi ) ,(2\pi , 3\pi) ,...$ (ส่วนฝั่งลบก็สมมาตรกับฝั่งบวก)

เพราะฉะนั้น แสดงว่า 4n+1 จุด เกิดได้กรณีเดียวคือ จุดสุดท้ายก่อนเส้นตรงพ้นจากกราฟ sin x นั้น กราฟเส้นตรงจะต้องสัมผัสกราฟ sin x พอดี ที่ $ x= x_n= 2n\pi + l_n \,\, ,0< l_n \leq \frac{\pi}{2} $

เนื่องจาก slope เส้นตรงดูจาก $ a_n $ ดังนั้น $ a_n = \cos x_n = \cos l_n $

เมื่อ n มากขึ้น จำนวนจุดตัดมากขึ้น ดังนั้น $a_n$ เป็นลำดับลด ส่งผลให้ $ l_n $ ลู่เข้า $\frac{\pi}{2}$

ในขณะเดียวกัน $ a_n = \frac{y_n}{x_n} = \frac{\sin x_n}{x_n} = \frac{\sin l_n}{2n\pi + l_n}$

ดังนั้น $ \lim_{n\rightarrow\infty} na_n =\lim_{n\rightarrow\infty}\frac{n\sin l_n}{2n\pi + l_n} = \lim_{n\rightarrow\infty}\frac{\sin l_n}{2\pi + \frac{l_n}{n}} = \frac{1}{2\pi}$

Note : $ \lim_{n\rightarrow\infty} \frac{l_n}{n} = 0 $ โดยใช้ squeezing theorem และพิจารณา

$ 0 < \frac{l_n}{n} \leq \frac{\pi /2}{n}$

N'Pon Jukku AuHu Jealyai 01 ธันวาคม 2012 15:14

ขอบคุณคุณ gon มากครับ
ได้มาแล้วลงในบอร์ดนี้นะครับ

gon 01 ธันวาคม 2012 16:10

อ้างอิง:

ข้อความเดิมเขียนโดยคุณ N'Pon Jukku AuHu Jealyai (ข้อความที่ 151376)
ขอบคุณคุณ gon มากครับ
ได้มาแล้วลงในบอร์ดนี้นะครับ

ผมได้ตัวข้อสอบฉบับจริงมาแล้วครับ แต่เป็นเวอร์ชันถ่ายเอกสารมา ซึ่งไม่ค่อยชัดเท่าไร

สงสัยว่าหมึกใกล้จะหมด :haha:

เดี๋ยวผมจะหาเวลาว่างสแกนภายในเวลาที่ผมว่าไว้ ;)

bell18 01 ธันวาคม 2012 20:13

สู้ๆครับพี่ gon :)


เวลาที่แสดงทั้งหมด เป็นเวลาที่ประเทศไทย (GMT +7) ขณะนี้เป็นเวลา 16:42

Powered by vBulletin® Copyright ©2000 - 2024, Jelsoft Enterprises Ltd.
Modified by Jetsada Karnpracha